Difference between revisions of "1994 AJHSME Problems/Problem 1"

(Created page with "==Problem== Which of the following is the largest? <math>\text{(A)}\ \dfrac{1}{3} \qquad \text{(B)}\ \dfrac{1}{4} \qquad \text{(C)}\ \dfrac{3}{8} \qquad \text{(D)}\ \dfrac{5}{12...")
 
Line 3: Line 3:
  
 
<math>\text{(A)}\ \dfrac{1}{3} \qquad \text{(B)}\ \dfrac{1}{4} \qquad \text{(C)}\ \dfrac{3}{8} \qquad \text{(D)}\ \dfrac{5}{12} \qquad \text{(E)}\ \dfrac{7}{24}</math>
 
<math>\text{(A)}\ \dfrac{1}{3} \qquad \text{(B)}\ \dfrac{1}{4} \qquad \text{(C)}\ \dfrac{3}{8} \qquad \text{(D)}\ \dfrac{5}{12} \qquad \text{(E)}\ \dfrac{7}{24}</math>
 +
 +
==Solution==
 +
 +
<math>\dfrac{3}{8}</math> and <math>\dfrac{5}{12}</math> are the only fractions where the numerator is close to half of the denominator. <math>\dfrac{3}{8}</math> is equal to .375 and <math>\dfrac{5}{12}</math> is equal to approximately .41. <math>\boxed{\text{(D)}}</math>

Revision as of 17:51, 14 November 2011

Problem

Which of the following is the largest?

$\text{(A)}\ \dfrac{1}{3} \qquad \text{(B)}\ \dfrac{1}{4} \qquad \text{(C)}\ \dfrac{3}{8} \qquad \text{(D)}\ \dfrac{5}{12} \qquad \text{(E)}\ \dfrac{7}{24}$

Solution

$\dfrac{3}{8}$ and $\dfrac{5}{12}$ are the only fractions where the numerator is close to half of the denominator. $\dfrac{3}{8}$ is equal to .375 and $\dfrac{5}{12}$ is equal to approximately .41. $\boxed{\text{(D)}}$